Question

2. (25 points). Rabi oscillations. Consider a spin-1/2 particle in a magnetic field B - Bo2 such that the spin eigenstates ar

0 0
Add a comment Improve this question Transcribed image text
Answer #1

ven 2ee Pobi o Cas (us hus ut91 e 2- ko。

Add a comment
Know the answer?
Add Answer to:
2. (25 points). Rabi oscillations. Consider a spin-1/2 particle in a magnetic field B - Bo2 such ...
Your Answer:

Post as a guest

Your Name:

What's your source?

Earn Coins

Coins can be redeemed for fabulous gifts.

Not the answer you're looking for? Ask your own homework help question. Our experts will answer your question WITHIN MINUTES for Free.
Similar Homework Help Questions
  • 3.16 For a spin-1/2 system undergoing Rabi oscillations, assume that the resonance condition holds. a) Solve...

    3.16 For a spin-1/2 system undergoing Rabi oscillations, assume that the resonance condition holds. a) Solve the differential equations for the coefficients α + (r). Use your results to find the transformed state vector |ψ(t)) and the state vector ψ(t), assuming the most general b) Verify that a π-pulse (wit-T) produces a complete spin flip. Calculate both the c) Assume that the interaction time is such that ω/-: π/2. Find the effect on the system d) Discuss the differences between...

  • 1. The aim of this problem set is to understand the dynamics of a spin-1/2 system...

    1. The aim of this problem set is to understand the dynamics of a spin-1/2 system in its full glory. Note that formally a spin-1/2 system and a qubit are equivalent hence, all what you will discover in this problem set will carry over to single qubits. Consider an electron spin (spin 1/2, magnetic moment gHB) interacting with a strong magnetic field Bo (0,0, B) in the z direction as well as with a much weaker magnetic field Brf =...

  • Problem 111.3. A spin 1/2 particle interacts with a nnagnetic field B = Boe through the...

    Problem 111.3. A spin 1/2 particle interacts with a nnagnetic field B = Boe through the Pauli interaction H-μσ. B where μ is the magnetic moment. The Pauli spin matrices are İ-(Oz,@yMwwhere the σί are T0 1 0-il The eigenstates for d, are the spinors 0 (a) (3 pts.) Suppose that at time t-0 the particle is in an eigenstate Xx corresponding to spin pointing along the positive z-axis. Find the eigenstatexz in terms of α and β. (b) (7...

  • 1. (30 points). Coupled spins. Spin-1/2 particles A and B evolve under the influence of the follo...

    1. (30 points). Coupled spins. Spin-1/2 particles A and B evolve under the influence of the following Hamiltonian (for simplicity takeh-1 so that energies are expressed in frequency units): We work in the uncoupled basis aba) Ib), where a,b E 0,1 and where states 0) (1)) correspond to single spins aligned (antialigned) with the z-direction. As we discussed in lecture, the eigenstates of the Hamiltonian are 100), 111), and 2-1/2 (101) 110)). a. We prepare the initial state t01). Since...

  • qm 2019.3 3. The Hamiltonian corresponding to the magnetic interaction of a spin 1/2 particle with...

    qm 2019.3 3. The Hamiltonian corresponding to the magnetic interaction of a spin 1/2 particle with charge e and mass m in a magnetic field B is À eB B. Ŝ, m where Ŝ are the spin angular momentum operators. You should make use of expres- sions for the spin operators that are given at the end of the question. (i) Write down the energy eigenvalue equation for this particle in a field directed along the y axis, i.e. B...

  • 6. Given the spin Hamiltonian of an electron in a magnetic field B-Bok, H--y5.B, find the...

    6. Given the spin Hamiltonian of an electron in a magnetic field B-Bok, H--y5.B, find the time evolution Unitary operator given as U(6) - exp(-()/N). Given that the initial quantum state is le (O) >= 0) find the state after a time t given as (10 points) () >= U(t)|(0) >

  • A spin-1 particle interacts with an external magnetic field B = B. The interaction Hamiltonian for the system is H = gB-S, where S-Si + Sỳ + SE is the spin operator. (Ignore all degrees of freedom ot...

    A spin-1 particle interacts with an external magnetic field B = B. The interaction Hamiltonian for the system is H = gB-S, where S-Si + Sỳ + SE is the spin operator. (Ignore all degrees of freedom other than spin.) (a) Find the spin matrices in the basis of the S. S eigenstates, |s, m)) . (Hint: Use the ladder operators, S -S, iS, and S_-S-iS,, and show first that s_ | 1,0-ћ /2 | 1.-1)) . Then use these...

  • 2 Two-level system Consider the time-dependent tion ihub = Hub Hamiltonian Schrödinger equa- for ...

    2 Two-level system Consider the time-dependent tion ihub = Hub Hamiltonian Schrödinger equa- for a two-level system with a (13) Use the ansatz ψ-ee(t)e-iwt/21e) + cg(t)ewt/21g) (14) for the state a) Derive the (exact) differential equations for ce(t),cg(t) b) Use a Fourier-series ansatz, ce- en einwptan ,eg Ση einWptbn. Show that the equations hold. Find m (consider the case separately) wWp c) Find an iterative procedure to solve these equations to higher and higher accuracy. Calcu- late the leading order...

  • Consider the state of a spin-1/2 particle 14) = v1o (31+z) + i] – z)) where...

    Consider the state of a spin-1/2 particle 14) = v1o (31+z) + i] – z)) where | z) are the eigenstates of the operator of the spin z-component $z. 1. Show that [V) is properly normalized, i.e. (W14) = 1. 2. Calculate the probability that a measurement of $x = 6x yields 3. Calculate the expectation value (Šx) for the state 14) and its dispersion ASx = V(@z) – ($()2. 4. Assume that the spin is placed in the magnetic...

  • The behavior of a spin- particle in a uniform magnetic field in the z-direction, , with...

    The behavior of a spin- particle in a uniform magnetic field in the z-direction, , with the Hamiltonian You found that the expectation value of the spin vector undergoes Larmor precession about the z axis. In this sense, we can view it as an analogue to a rotating coin, choosing the eigenstate with eigenvalue to represent heads and the eigenstate with eigenvalue to represent tails. Under time-evolution in the magnetic field, these eigenstates will “rotate” between each other. (a) Suppose...

ADVERTISEMENT
Free Homework Help App
Download From Google Play
Scan Your Homework
to Get Instant Free Answers
Need Online Homework Help?
Ask a Question
Get Answers For Free
Most questions answered within 3 hours.
ADVERTISEMENT
ADVERTISEMENT
ADVERTISEMENT